Warum ist π0→γ→e−e+π0→γ→e−e+\pi^0 \rightarrow \gamma \rightarrow e^-e^+ verboten, aber π+→W+→e+νeπ+→W+→e+νe \pi^+\rightarrow W^+ \rightarrow e^+ \nu_e erlaubt?

Ich bin etwas verwirrt, warum der Zerfall π 0 γ e e + ist verboten. Eine naive Vermutung würde sagen, dass das Zwischenphoton γ einen Spin 1 hat, hat das anfängliche Pion einen Spin 0 daher verletzt dies die Spinerhaltung. Allerdings mit der gleichen Begründung π + W + e + v e wäre verboten, ist es aber nicht. Deshalb kann ich nicht verstehen, warum wir nicht haben können π 0 γ e e + doch noch haben π + W + e + v e . Kann bitte jemand erklären? (ps alle Zwischenpartikel sollten als virtuell angesehen werden).

Es gibt ein echtes Problem mit Ihrer Notation. 1-Körper zerfällt wie π 0 γ Und π 0 W + sind kinematisch verboten, da sie nicht sowohl Energie als auch Impuls erhalten können. Das Photon ist masselos, also können Sie es nicht haben γ e + e entweder.
@dukwon Entschuldigung, ich habe alle Zwischenpartikel als virtuell angesehen.
Ihre Frage betrifft also Feynman-Diagramme, die zu Zerfallsamplituden beitragen, nicht die Zerfälle selbst. Es ist wichtig, die Falle zu vermeiden, zu wörtlich an Feynman-Diagramme zu denken. Dies scheint ein Duplikat von physical.stackexchange.com/q/233076 zu sein

Antworten (2)

Die QCD und die QED selbst bewahren die Parität. Die Schlussfolgerung dieser Aussage ist, dass alle entsprechenden effektiven Eckpunkte die Parität bewahren müssen. Die einzige Kupplung von π 0 Zu γ Erhaltung der Parität ist

L π 0 π 0 Λ ϵ μ v a β F μ v F a β ,
was deinen Verfallsprozess nicht zulässt π 0 γ e e + . Um dies zu verstehen, beachten Sie, dass die Pion-Felder Pseudo-Skalare sind, während das Photonenfeld das Vektorfeld ist, während der Levi-Civita-Tensor ϵ μ v a β ist der Pseudo-Tensor.

Es ist jedoch möglich, den effektiven Scheitelpunkt zu konstruieren, der den Zerfall ermöglicht π 0 e + e durch Z -Boson, dh durch schwache Wechselwirkungen. Der Grund ist, dass sie direkt die Parität verletzen. Daher ist es möglich, einen phänomenologischen paritätsverletzenden niedrigdimensionalen effektiven Interaktionsvertex zu konstruieren

L π 0 ' Λ ' μ π 0 Z μ ,
Zerfallsprozess ermöglichen π 0 Z e + e .

Aus dem gleichen Grund ist es einfach, paritätsverletzende Eckpunkte zu konstruieren

L π + = Λ ~ μ π + W μ +  hc ,
Ihren Verfallsprozess zulassen π + W + l + v l .

Hallo danke für deine Antwort, in meinem Vorlesungsskript gibt es ein Beispiel für einen Zerfall der J / ψ Partikel: J / ψ γ Q Q ¯ nach dem, was du hier gesagt hast, ist dieser zerfall auch verboten - richtig?
@Quantumspaghettification: die Drehung von J / ψ Meson ist eins, was den effektiven Scheitelpunkt ermöglicht ϵ μ v a β F μ v J / ψ F a β γ .
Ok, es ist etwas komplizierter, als zu sagen, dass wir den "Winkelimpuls" an jedem Scheitelpunkt erhalten müssen, oder wir brauchen die "Parität", um erhalten zu bleiben - wir brauchen tatsächlich die richtige Kombination, damit wir einen zulässigen effektiven Scheitelpunkt haben? Gibt es allgemeine Kombinationen von Spin und Parität, die Ihnen immer einen zulässigen effektiven Scheitelpunkt geben?
@Quantumspaghettification: Ja, durch Kombinieren der Lorentz-Invarianz und der Paritätsinvarianz ist es nicht schwer, alle möglichen effektiven Scheitelpunkte zu konstruieren.

Ihre Frage könnte verallgemeinert werden: Der Zerfall von Pionen auf eine ungerade Anzahl von Photonen ist verboten. Das ist eine Schlussfolgerung aus Furrys Theorem: Eine gerade Anzahl von Photonen ist einer ungeraden Anzahl von Photonen verboten. Hier ist die einzige Annahme, dass eine starke Wechselwirkung und EM eine Ladungskonjugationssymmetrie verarbeiten.

Da die Ladungskonjugation bei der schwachen Wechselwirkung keine Symmetrie mehr ist, wird beim zweiten Prozess der Zerfallsprozess zugelassen.